Anda di halaman 1dari 54

Cards

Front

Back

1. The nurse assesses a patient with shortness of breath for


evidence of long-standing hypoxemia by inspecting: A. Chest
excursion B. Spinal curvatures C. The respiratory pattern D.
The fingernail and its base

D. The fingernail and its base Clubbing, a sign of longstanding hypoxemia, is evidenced by an increase in the
angle between the base of the nail and the fingernail to 180
degrees or more, usually accompanied by an increase in
the depth, bulk, and sponginess of the end of the finger.

2. The nurse is caring for a patient with COPD and


pneumonia who has an order for arterial blood gases to be
drawn. Which of the following is the minimum length of time
the nurse should plan to hold pressure on the puncture site?
A. 2 minutes B. 5 minutes C. 10 minutes D. 15 minutes

B. 5 minutes Following obtaining an arterial blood gas, the


nurse should hold pressure on the puncture site for 5
minutes by the clock to be sure that bleeding has stopped.
An artery is an elastic vessel under higher pressure than
veins, and significant blood loss or hematoma formation
could occur if the time is insufficient.

3. The nurse notices clear nasal drainage in a patient newly


admitted with facial trauma, including a nasal fracture. The
nurse should: A. test the drainage for the presence of
glucose. B. suction the nose to maintain airway clearance. C.
document the findings and continue monitoring. D. apply a
drip pad and reassure the patient this is normal.

A. test the drainage for the presence of glucose. Clear


nasal drainage suggests leakage of cerebrospinal fluid
(CSF). The drainage should be tested for the presence of
glucose, which would indicate the presence of CSF.

4. When caring for a patient who is 3 hours postoperative


laryngectomy, the nurse's highest priority assessment would
be: A. Airway patency B. Patient comfort C. Incisional
drainage D. Blood pressure and heart rate

A. Airway patency Remember ABCs with prioritization.


Airway patency is always the highest priority and is
essential for a patient undergoing surgery surrounding the
upper respiratory system.

Front

Back

5. When initially teaching a patient the supraglottic swallow


following a radical neck dissection, with which of the following
foods should the nurse begin? A. Cola B. Applesauce C.
French fries D. White grape juice

A. ColaWhen learning the supraglottic swallow, it may be


helpful to start with carbonated beverages because the
effervescence provides clues about the liquid's position.
Thin, watery fluids should be avoided because they are
difficult to swallow and increase the risk of aspiration.
Nonpourable pureed foods, such as applesauce, would
decrease the risk of aspiration, but carbonated beverages
are the better choice to start with.

6. The nurse is caring for a patient admitted to the hospital


with pneumonia. Upon assessment, the nurse notes a
temperature of 101.4 F, a productive cough with yellow
sputum and a respiratory rate of 20. Which of the following
nursing diagnosis is most appropriate based upon this
assessment? A. Hyperthermia related to infectious illness B.
Ineffective thermoregulation related to chilling C. Ineffective
breathing pattern related to pneumonia D. Ineffective airway
clearance related to thick secretions

A. Hyperthermia related to infectious illness Because the


patient has spiked a temperature and has a diagnosis of
pneumonia, the logical nursing diagnosis is hyperthermia
related to infectious illness. There is no evidence of a chill,
and her breathing pattern is within normal limits at 20
breaths per minute. There is no evidence of ineffective
airway clearance from the information given because the
patient is expectorating sputum.

7. Which of the following physical assessment findings in a


patient with pneumonia best supports the nursing diagnosis
of ineffective airway clearance? A. Oxygen saturation of 85%
B. Respiratory rate of 28 C. Presence of greenish sputum D.
Basilar crackles

D. Basilar crackles The presence of adventitious breath


sounds indicates that there is accumulation of secretions in
the lower airways. This would be consistent with a nursing
diagnosis of ineffective airway clearance because the
patient is retaining secretions.

8. Which of the following clinical manifestations would the


nurse expect to find during assessment of a patient admitted
with pneumococcal pneumonia? A. Hyperresonance on

C. Increased vocal fremitus on palpation. A typical physical


examination finding for a patient with pneumonia is
increased vocal fremitus on palpation. Other signs of

Front

Back

percussion B. Fine crackles in all lobes on auscultation C.


Increased vocal fremitus on palpation D. Vesicular breath
sounds in all lobes

pulmonary consolidation include dullness to percussion,


bronchial breath sounds, and crackles in the affected area.

9. Which of the following nursing interventions is of the


highest priority in helping a patient expectorate thick
secretions related to pneumonia? A. Humidify the oxygen as
able B. Increase fluid intake to 3L/day if tolerated. C.
Administer cough suppressant q4hr. D. Teach patient to
splint the affected area.

B. Increase fluid intake to 3L/day if tolerated. Although


several interventions may help the patient expectorate
mucus, the highest priority should be on increasing fluid
intake, which will liquefy the secretions so that the patient
can expectorate them more easily. Humidifying the oxygen
is also helpful, but is not the primary intervention. Teaching
the patient to splint the affected area may also be helpful,
but does not liquefy the secretions so that they can be
removed.

10. During discharge teaching for a 65-year-old patient with


emphysema and pneumonia, which of the following vaccines
should the nurse recommend the patient receive? A. S.
aureus B. H. influenzae C. Pneumococcal D. Bacille
Calmette-Gurin (BCG)

C. Pneumococcal The pneumococcal vaccine is important


for patients with a history of heart or lung disease,
recovering from a severe illness, age 65 or over, or living in
a long-term care facility.

11. The nurse evaluates that discharge teaching for a patient


hospitalized with pneumonia has been most effective when
the patient states which of the following measures to prevent
a relapse? A. "I will increase my food intake to 2400 calories
a day to keep my immune system well." B. "I must use home
oxygen therapy for 3 months and then will have a chest x-ray
to reevaluate." C. "I will seek immediate medical treatment
for any upper respiratory infections." D. "I should continue to

D. "I should continue to do deep-breathing and coughing


exercises for at least 6 weeks." It is important for the patient
to continue with coughing and deep breathing exercises for
6 to 8 weeks until all of the infection has cleared from the
lungs. A patient should seek medical treatment for upper
respiratory infections that persist for more than 7 days.
Increased fluid intake, not caloric intake, is required to
liquefy secretions. Home O2 is not a requirement unless

Front

Back

do deep-breathing and coughing exercises for at least 6


weeks."

the patient's oxygenation saturation is below normal.

12. After admitting a patient to the medical unit with a


diagnosis of pneumonia, the nurse will verify that which of the
following physician orders have been completed before
administering a dose of cefotetan (Cefotan) to the patient? A.
Serum laboratory studies ordered for AM B. Pulmonary
function evaluation C. Orthostatic blood pressures D. Sputum
culture and sensitivity

D. Sputum culture and sensitivityThe nurse should ensure


that the sputum for culture and sensitivity was sent to the
laboratory before administering the cefotetan. It is important
that the organisms are correctly identified (by the culture)
before their numbers are affected by the antibiotic; the test
will also determine whether the proper antibiotic has been
ordered (sensitivity testing). Although antibiotic
administration should not be unduly delayed while waiting
for the patient to expectorate sputum, all of the other
options will not be affected by the administration of
antibiotics.

22. The nurse is assigned to care for a patient who has


anxiety and an exacerbation of asthma. Which of the
following is the primary reason for the nurse to carefully
inspect the chest wall of this patient? A. Observe for signs of
diaphoresis B. Allow time to calm the patient C. Monitor the
patient for bilateral chest expansion D. Evaluate the use of
intercostal muscles

D. Evaluate the use of intercostal muscles The nurse


physically inspects the chest wall to evaluate the use of
intercostal (accessory) muscles, which gives an indication
of the degree of respiratory distress experienced by the
patient.

13. Which of the following nursing interventions is most


appropriate to enhance oxygenation in a patient with
unilateral malignant lung disease? A. Positioning patient on
right side. B. Maintaining adequate fluid intake C. Performing
postural drainage every 4 hours D. Positioning patient with

D. Positioning patient with good lung down Therapeutic


positioning identifies the best position for the patient
assuring stable oxygenation status. Research indicates that
positioning the patient with the unaffected lung (good lung)
dependent best promotes oxygenation in patients with

Front

Back

good lung down

unilateral lung disease. For bilateral lung disease, the right


lung down has best ventilation and perfusion. Increasing
fluid intake and performing postural drainage will facilitate
airway clearance, but positioning is most appropriate to
enhance oxygenation.

14. A 71-year-old patient is admitted with acute respiratory


distress related to cor pulmonale. Which of the following
nursing interventions is most appropriate during admission of
this patient? A. Delay any physical assessment of the patient
and review with the family the patients history of respiratory
problems. B. Perform a comprehensive health history with
the patient to review prior respiratory problems. C. Perform a
physical assessment of the respiratory system and ask
specific questions related to this episode of respiratory
distress. D. Complete a full physical examination to
determine the effect of the respiratory distress on other body
functions.

C. Perform a physical assessment of the respiratory system


and ask specific questions related to this episode of
respiratory distress.Because the patient is having
respiratory difficulty, the nurse should ask specific
questions about this episode and perform a physical
assessment of this system. Further history taking and
physical examination of other body systems can proceed
once the patient's acute respiratory distress is being
managed.

15. When planning appropriate nursing interventions for a


patient with metastatic lung cancer and a 60-pack-year
history of cigarette smoking, the nurse recognizes that the
smoking has most likely decreased the patient's underlying
respiratory defenses because of impairment of which of the
following? A. Reflex bronchoconstriction B. Ability to filter
particles from the air C. Cough reflex D. Mucociliary
clearance

D. Mucociliary clearance Smoking decreases the ciliary


action in the tracheobronchial tree, resulting in impaired
clearance of respiratory secretions, chronic cough, and
frequent respiratory infections.

Front

Back

16. While ambulating a patient with metastatic lung cancer,


the nurse observes a drop in oxygen saturation from 93% to
86%. Which of the following nursing interventions is most
appropriate based upon these findings? A. Continue with
ambulation as this is a normal response to activity. B. Move
the oximetry probe from the finger to the earlobe for more
accurate monitoring during activity. C. Obtain a physicians
order for supplemental oxygen to be used during ambulation
and other activity. D. Obtain a physicians order for arterial
blood gas determinations to verify the oxygen saturation.

C. Obtain a physicians order for supplemental oxygen to


be used during ambulation and other activity. An oxygen
saturation level that drops below 90% with activity indicates
that the patient is not tolerating the exercise and needs to
have supplemental oxygen applied.

17. The nurse is caring for a 73-year-old patient who


underwent a left total knee arthroplasty. On the third
postoperative day, the patient complains of shortness of
breath, slight chest pain, and that "something is wrong."
Temperature is 98.4o F, blood pressure 130/88, respirations
36, and oxygen saturation 91% on room air. Which of the
following should the nurse first suspect as the etiology of this
episode? A. Septic embolus from the knee joint B. Pulmonary
embolus from deep vein thrombosis C. New onset of angina
pectoris D. Pleural effusion related to positioning in the
operating room

B. Pulmonary embolus from deep vein thrombosis The


patient presents the classic symptoms of pulmonary
embolus: acute onset of symptoms, tachypnea, shortness
of breath, and chest pain.

18. In the case of pulmonary embolus from deep vein


thrombosis, which of the following actions should the nurse
take first? A. Notify the physician. B. Administer a
nitroglycerin tablet sublingually. C. Conduct a thorough
assessment of the chest pain. D. Sit the patient up in bed as

D. Sit the patient up in bed as tolerated and apply


oxygen.The patient's clinical picture is consistent with
pulmonary embolus, and the first action the nurse takes
should be to assist the patient. For this reason, the nurse
should sit the patient up as tolerated and apply oxygen

Front

Back

tolerated and apply oxygen.

before notifying the physician.

19. The nurse is caring for a postoperative patient with


sudden onset of respiratory distress. The physician orders a
STAT ventilation-perfusion scan. Which of the following
explanations should the nurse provide to the patient about
the procedure? A. This test involves injection of a
radioisotope to outline the blood vessels in the lungs,
followed by inhalation of a radioisotope gas. B. This test will
use special technology to examine cross sections of the
chest with use of a contrast dye. C. This test will use
magnetic fields to produce images of the lungs and chest. D.
This test involves injecting contrast dye into a blood vessel to
outline the blood vessels of the lungs.

A. This test involves injection of a radioisotope to outline


the blood vessels in the lungs, followed by inhalation of a
radioisotope gas.A ventilation-perfusion scan has two parts.
In the perfusion portion, a radioisotope is injected into the
blood and the pulmonary vasculature is outlined. In the
ventilation part, the patient inhales a radioactive gas that
outlines the alveoli.

20. During assessment of a 45-year-old patient with asthma,


the nurse notes wheezing and dyspnea. The nurse interprets
that these symptoms are related to which of the following
pathophysiologic changes? A. Laryngospasm B.
Overdistention of the alveoli C. Narrowing of the airway D.
Pulmonary edema

C. Narrowing of the airwayNarrowing of the airway leads to


reduced airflow, making it difficult for the patient to breathe
and producing the characteristic wheezing.

21. A 45-year-old man with asthma is brought to the


emergency department by automobile. He is short of breath
and appears frightened. During the initial nursing
assessment, which of the following clinical manifestations
might be present as an early symptom during an
exacerbation of asthma? A. Anxiety B. Cyanosis C.

A. Anxiety An early symptom during an asthma attack is


anxiety because he is acutely aware of the inability to get
sufficient air to breathe. He will be hypoxic early on with
decreased PaCO2 and increased pH as he is
hyperventilating.

Front

Back

Hypercapnia D. Bradycardia

23. Which of the following positions is most appropriate for


the nurse to place a patient experiencing an asthma
exacerbation? A. Supine B. Lithotomy C. High-Fowlers D.
Reverse Trendelenburg

C. High-FowlersThe patient experiencing an asthma attack


should be placed in high-Fowler's position to allow for
optimal chest expansion and enlist the aid of gravity during
inspiration.

24. The nurse is caring for a patient with an acute


exacerbation of asthma. Following initial treatment, which of
the following findings indicates to the nurse that the patient's
respiratory status is improving? A. Wheezing becomes louder
B. Vesicular breath sounds decrease C. Aerosol
bronchodilators stimulate coughing D. The cough remains
nonproductive

A. Wheezing becomes louder The primary problem during


an exacerbation of asthma is narrowing of the airway and
subsequent diminished air exchange. As the airways begin
to dilate, wheezing gets louder because of better air
exchange.

25. The nurse identifies the nursing diagnosis of activity


intolerance for a patient with asthma. The nurse assesses for
which of the following etiologic factor for this nursing
diagnosis in patients with asthma? A. Anxiety and
restlessness B. Effects of medications C. Fear of suffocation
D. Work of breathing

D. Work of breathingWhen the patient does not have


sufficient gas exchange to engage in activity, the etiologic
factor is often the work of breathing. When patients with
asthma do not have effective respirations, they use all
available energy to breathe and have little left over for
purposeful activity.

26. The nurse is assigned to care for a patient in the


emergency department admitted with an exacerbation of
asthma. The patient has received a -adrenergic
bronchodilator and supplemental oxygen. If the patient's
condition does not improve, the nurse should anticipate

B. Systemic corticosteroids Systemic corticosteroids speed


the resolution of asthma exacerbations and are indicated if
the initial response to the -adrenergic bronchodilator is
insufficient.

Front

Back

which of the following is likely to be the next step in


treatment? A. Pulmonary function testing B. Systemic
corticosteroids C. Biofeedback therapy D. Intravenous fluids

27. A patient with acute exacerbation of COPD needs to


receive precise amounts of oxygen. Which of the following
types of equipment should the nurse prepare to use? A.
Venturi mask B. Partial non-rebreather mask C. Oxygen tent
D. Nasal cannula

A. Venturi mask The Venturi mask delivers precise


concentrations of oxygen and should be selected whenever
this is a priority concern. The other methods are less
precise in terms of amount of oxygen delivered.

28. While teaching a patient with asthma about the


appropriate use of a peak flow meter, the nurse instructs the
patient to do which of the following? A. Use the flow meter
each morning after taking medications to evaluate their
effectiveness. B. Empty the lungs and then inhale quickly
through the mouthpiece to measure how fast air can be
inhaled. C. Keep a record of the peak flow meter numbers if
symptoms of asthma are getting worse. D. Increase the
doses of the long-term control medication if the peak flow
numbers decrease.

C. Keep a record of the peak flow meter numbers if


symptoms of asthma are getting worse. It is important to
keep track of peak flow readings daily and when the
patient's symptoms are getting worse. The patient should
have specific directions as to when to call the physician
based on personal peak flow numbers. Peak flow is
measured by exhaling into the meters and should be
assessed before and after medications to evaluate their
effectiveness.

56. When admitting a 45-year-old female with a diagnosis of


pulmonary embolism, the nurse will assess the patient for
which of the following risk factors? (Select all that apply.) A.
Obesity B. Pneumonia C. Hypertension D. Cigarette smoking

A,C,D Research has demonstrated an increased risk of


pulmonary embolism in women associated with obesity,
heavy cigarette smoking, and hypertension. Other risk
factors include immobilization, surgery within the last 3
months, stroke, history of DVT, and malignancy.

Front

Back

29. The physician has prescribed salmeterol (Serevent) for a


patient with asthma. In reviewing the use of dry powder
inhalers (DPIs) with the patient, the nurse should provide
which of the following instructions? A. Close lips tightly
around the mouthpiece and breathe in deeply and quickly.
B. To administer a DPI, you must use a spacer that holds
the medicine so that you can inhale it. C. Hold the inhaler
several inches in front of your mouth and breathe in slowly,
holding the medicine as long as possible. D. You will know
you have correctly used the DPI when you taste or sense the
medicine going into your lungs.

A. Close lips tightly around the mouthpiece and breathe in


deeply and quickly. Dry powder inhalers do not require
spacer devices. The patient should be instructed to breathe
in deeply and quickly to ensure medicine moves down
deeply into lungs. The patient may not taste or sense the
medicine going into the lungs.

30. The nurse determines that a patient is experiencing


common adverse effects from the inhaled corticosteroid
beclomethasone (Beclovent) after noting which of the
following? A. Adrenocortical dysfunction and hyperglycemia
B. Elevation of blood glucose and calcium levels C.
Oropharyngeal candidiasis and hoarseness D. Hypertension
and pulmonary edema

C. Oropharyngeal candidiasis and hoarseness


Oropharyngeal candidiasis and hoarseness are common
adverse effects from the use of inhaled corticosteroids
because the medication can lead to overgrowth of
organisms and local irritation if the patient does not rinse
the mouth following each dose.

31. The nurse determines that the patient understood


medication instructions about the use of a spacer device
when taking inhaled medications after hearing the patient
state which of the following as the primary benefit? A. Now I
will not need to breathe in as deeply when taking the inhaler
medications. B. This device will make it so much easier and
faster to take my inhaled medications. C. I will pay less for
medication because it will last longer. D. More of the

D. More of the medication will get down into my lungs to


help my breathing. A spacer assists more medication to
reach the lungs, with less being deposited in the mouth and
the back of the throat.

Front

Back

medication will get down into my lungs to help my breathing.

32. Which of the following test results identify that a patient


with an asthma attack is responding to treatment? A. A
decreased exhaled nitric oxide B. An increase in CO2 levels
C. A decrease in white blood cell count D. An increase in
serum bicarbonate levels

A. A decreased exhaled nitric oxide. Nitric oxide levels are


increased in the breath of people with asthma. A decrease
in the exhaled nitric oxide concentration suggests that the
treatment may be decreasing the lung inflammation
associated with asthma.

33. The nurse determines that the patient is not experiencing


adverse effects of albuterol (Proventil) after noting which of
the following patient vital signs? A. Oxygen saturation 96% B.
Respiratory rate of 18 C. Temperature of 98.4 F D. Pulse
rate of 76

D. Pulse rate of 76 Albuterol is a 2-agonist that can


sometimes cause adverse cardiovascular effects. These
would include tachycardia and angina. A pulse rate of 76
indicates that the patient did not experience tachycardia as
an adverse effect.

34. The patient has an order for each of the following


inhalers. Which of the following should the nurse offer to the
patient at the onset of an asthma attack? A. Albuterol
(Proventil) B. Beclomethasone (Beclovent) C. Ipratropium
bromide (Atrovent) D. Salmeterol (Serevent)

A. Albuterol (Proventil) Albuterol is a short-acting


bronchodilator that should initially be given when the patient
experiences an asthma attack.

35. The nurse who has administered a first dose of oral


prednisone (Deltasone) to the patient with asthma writes on
the care plan to begin monitoring which of the following
patient parameters? A. Intake and output B. Bowel sounds C.
Apical pulse D. Deep tendon reflexes

A. Intake and output Corticosteroids such as prednisone


can lead to fluid retention. For this reason, it is important to
monitor the patient's intake and output.

Front

Back

36. The nurse is assisting a patient to learn selfadministration of beclomethasone two puffs inhalation q6hr.
The nurse explains that the best way to prevent oral infection
while taking this medication is to do which of the following as
part of the self-administration techniques? A. Chew a hard
candy before the first puff of medication. B. Ask for a breath
mint following the second puff of medication. C. Rinse the
mouth with water before each puff of medication. D. Rinse
the mouth with water following the second puff of medication.

D. Rinse the mouth with water following the second puff of


medication. The patient should rinse the mouth with water
following the second puff of medication to reduce the risk of
fungal overgrowth and oral infection.

37. The nurse is scheduled to give a dose of salmeterol by


metered dose inhaler (MDI). The nurse would administer the
right drug by selecting the inhaler with which of the following
trade names? A. Vanceril B. Serevent C. AeroBid D. Atrovent

B. Serevent The trade or brand name for salmeterol, an


adrenergic bronchodilator, is Serevent.

38. The nurse is evaluating whether a patient understands


how to safely determine whether a metered dose inhaler is
empty. The nurse interprets that the patient understands this
important information to prevent medication underdosing
when the patient describes which method to check the
inhaler? A. Place it in water to see if it floats. B. Shake the
canister while holding it next to the ear. C. Check the
indicator line on the side of the canister. D. Keep track of the
number of inhalations used.

D. Keep track of the number of inhalations used. It is no


longer appropriate to see if a canister floats in water or not
as research has demonstrated this is not accurate. The
best method to determine when to replace an inhaler is by
knowing the maximum puffs available per MDI and then
replacing when those inhalations have been used.

39. The nurse is scheduled to give a dose of ipratropium


bromide by metered dose inhaler. The nurse would
administer the right drug by selecting the inhaler with which

D. Atrovent The trade or brand name for ipratropium


bromide, an anticholinergic medication, is Atrovent.

Front

Back

of the following trade names? A. Vanceril B. Pulmicort C.


AeroBid D. Atrovent

40. The patient has an order for albuterol 5 mg via nebulizer.


Available is a solution containing 2 mg/ml. How many
milliliters should the nurse use to prepare the patient's dose?
A. 0.2 B. 2.5 C. 3.75 D. 5.0

B. 2.5

41. When planning patient teaching about emphysema, the


nurse understands that the symptoms of emphysema are
caused by which of the following? A. Hypertrophy and
hyperplasia of goblet cells in the bronchi B. Collapse and
hypoventilation of the terminal respiratory unit C. An
overproduction of the antiprotease alpha1-antitrypsin D.
Hyperinflation of alveoli and destruction of alveolar walls

D. Hyperinflation of alveoli and destruction of alveolar walls


In emphysema, there are structural changes that include
hyperinflation of alveoli, destruction of alveolar walls,
destruction of alveolar capillary walls, narrowing of small
airways, and loss of lung elasticity.

42. The patient has an order for albuterol 5 mg via nebulizer.


Available is a solution containing 1 mg/ml. How many
milliliters should the nurse use to prepare the patient's dose?
A. 0.2 B. 2.5 C. 3.75 D. 5.0

D. 5.0

43. The nurse evaluates that nursing interventions to


promote airway clearance in a patient admitted with COPD
are successful based on which of the following findings? A.
Absence of dyspnea B. Improved mental status C. Effective
and productive coughing D. PaO2 within normal range for the

C. Effective and productive coughing The issue of the


question is airway clearance, which is most directly
evaluated as successful if the patient can engage in
effective and productive coughing.

Front

Back

patient

44. When caring for a patient with COPD, the nurse identifies
a nursing diagnosis of imbalanced nutrition less than body
requirements after noting a weight loss of 30 lb. Which of the
following would be an appropriate intervention to add to the
plan of care for this patient? A. Teach the patient to use
frozen meals at home that can be microwaved. B. Provide a
high-calorie, high-carbohydrate, nonirritating, frequent
feeding diet. C. Order fruits and fruit juices to be offered
between meals. D. Order a high-calorie, high-protein diet with
six small meals a day.

D. Order a high-calorie, high-protein diet with six small


meals a day.Because the patient with COPD needs to use
greater energy to breathe, there is often decreased oral
intake because of dyspnea. A full stomach also impairs the
ability of the diaphragm to descend during inspiration,
interfering with the work of breathing. Finally, the
metabolism of a high carbohydrate diet yields large
amounts of CO2, which may lead to acidosis in patients
with pulmonary disease. For these reasons, the patient with
emphysema should take in a high-calorie, high-protein diet,
eating six small meals per day.

45. The nurse reviews pursed lip breathing with a patient


newly diagnosed with emphysema. The nurse reinforces that
this technique will assist respiration by which of the following
mechanisms? A. Preventing bronchial collapse and air
trapping in the lungs during exhalation B. Increasing the
respiratory rate and giving the patient control of respiratory
patterns C. Loosening secretions so that they may be
coughed up more easily D. Promoting maximal inhalation for
better oxygenation of the lungs

A. Preventing bronchial collapse and air trapping in the


lungs during exhalation The focus of pursed lip breathing is
to slow down the exhalation phase of respiration, which
decreases bronchial collapse and subsequent air trapping
in the lungs during exhalation.

46. Nursing assessment findings of jugular vein distention


and pedal edema would be indicative of which of the
following complications of emphysema? A. Acute respiratory
failure B. Pulmonary edema caused by left-sided heart failure

C. Fluid volume excess secondary to cor pulmonale Cor


pulmonale is a right-sided heart failure caused by
resistance to right ventricular outflow due to lung disease.
With failure of the right ventricle, the blood emptying into

Front

Back

C. Fluid volume excess secondary to cor pulmonale D.


Secondary respiratory infection

the right atrium and ventricle would be slowed, leading to


jugular venous distention and pedal edema.

47. A patient has been receiving oxygen per nasal cannula


while hospitalized for COPD. The patient asks the nurse
whether oxygen use will be needed at home. Which of the
following would be the most appropriate response by the
nurse? A. Long-term home oxygen therapy should be used
to prevent respiratory failure. B. Oxygen will be needed
when your oxygen saturation drops to 88% and you have
symptoms of hypoxia. C. Long-term home oxygen therapy
should be used to prevent heart problems related to
emphysema. D. Oxygen will not be needed until or unless
you are in the terminal stages of this disease.

B. Oxygen will be needed when your oxygen saturation


drops to 88% and you have symptoms of hypoxia.Longterm oxygen therapy in the home should be considered
when the oxygen saturation is 88% or less and the patient
has signs of tissue hypoxia, such as cor pulmonale,
erythrocytosis, or impaired mental status.

48. Before discharge, the nurse discusses activity levels with


a 61-year-old patient with COPD and pneumonia. Which of
the following exercise goals is most appropriate once the
patient is fully recovered from this episode of illness? A.
Slightly increase activity over the current level. B. Walk for 20
minutes a day, keeping the pulse rate less than 130 beats
per minute. C. Limit exercise to activities of daily living to
conserve energy. D. Swim for 10 min/day, gradually
increasing to 30 min/day.

B. Walk for 20 minutes a day, keeping the pulse rate less


than 130 beats per minute. The patient will benefit from
mild aerobic exercise that does not stress the
cardiorespiratory system. The patient should be
encouraged to walk for 20 min/day, keeping the pulse rate
less than 75% to 80% of maximum heart rate (220 minus
patient's age).

49. The nurse evaluates that a patient is experiencing the


expected beneficial effects of ipratropium (Atrovent) after
noting which of the following assessment findings? A.

A. Increased peak flow readings. Ipratropium is a


bronchodilator that should lead to increased PEFRs.

Front

Back

Increased peak flow readings B. Increased level of


consciousness C. Decreased sputum production D.
Increased respiratory rate

50. The nurse is teaching a patient how to self-administer


ipratropium (Atrovent) via a metered dose inhaler. Which of
the following instructions given by the nurse is most
appropriate to help the patient learn proper inhalation
technique? A. Avoid shaking the inhaler before use. B.
Breathe out slowly before positioning the inhaler. C. After
taking a puff, hold the breath for 30 seconds before
exhaling. D. Using a spacer should be avoided for this type
of medication.

B. Breathe out slowly before positioning the inhaler. It is


important to breathe out slowly before positioning the
inhaler. This allows the patient to take a deeper breath
while inhaling the medication thus enhancing the
effectiveness of the dose.

51. Which of the following statements made by a patient with


COPD indicates a need for further education regarding the
use of an ipratropium inhaler? A. I should rinse my mouth
following the two puffs to get rid of the bad taste. B. I should
wait at least 1 to 2 minutes between each puff of the inhaler.
C. If my breathing gets worse, I should keep taking extra
puffs of the inhaler until I can breathe more easily. D.
Because this medication is not fast-acting, I cannot use it in
an emergency if my breathing gets worse.

C. If my breathing gets worse, I should keep taking extra


puffs of the inhaler until I can breathe more easily. The
patient should not take extra puffs of the inhaler at will to
make breathing easier. Excessive treatment could trigger
paradoxical bronchospasm, which would worsen the
patient's respiratory status.

52. When assessing a patient's sleep-rest pattern related to


respiratory health, the nurse would ask if the patient: (Select
all that apply.) A. Has trouble falling asleep B. Awakens
abruptly during the night C. Sleeps more than 8 hours per

A,B,D The patient with sleep apnea may have insomnia


and/or abrupt awakenings. Patients with cardiovascular
disease (e.g., heart failure that may affect respiratory
health) may need to sleep with the head elevated on

Front

Back

night D. Has to sleep with the head elevated

several pillows (orthopnea). Sleeping more than 8 hours


per night is not indicative of impaired respiratory health.

53. A patient is being discharged from the emergency


department after being treated for epistaxis. In teaching the
family first aid measures in the event the epistaxis would
recur, which of the following measures would the nurse
suggest? (Select all that apply.) A. Tilt patients head
backwards B. Apply ice compresses to the nose C. Pinch the
entire soft lower portion of the nose D. Partially insert a small
gauze pad into the bleeding nostril

B,C,D First aid measures to control epistaxis includes


placing the patient in a sitting position, leaning forward.
Tilting the head back does not stop the bleeding, but rather
allows the blood to enter the nasopharynx, which could
result in aspiration or nausea/vomiting from swallowing
blood. All of the other options are appropriate first aid
treatment of epistaxis.

54. To promote airway clearance in a patient with


pneumonia, the nurse instructs the patient to do which of the
following? (Select all that apply.) A. Splint the chest when
coughing B. Maintain a semi-Fowler's position C. Maintain
adequate fluid intake D. Instruct patient to cough at end of
exhalation

A,C,D The nurse should instruct the patient to splint the


chest while coughing. This will reduce discomfort and allow
for a more effective cough. Maintaining adequate fluid
intake liquefies secretions, allowing easier expectoration.
Coughing at the end of exhalation promotes a more
effective cough. The patient should be positioned in an
upright sitting position (high-Fowler's) with head slightly
flexed.

55. During admission of a patient diagnosed with nonsmall


cell carcinoma of the lung, the nurse questions the patient
related to a history of which of the following risk factors for
this type of cancer? (Select all that apply.) A. Asbestos
exposure B. Cigarette smoking C. Exposure to uranium D.
Chronic interstitial fibrosis

A,B,C Non-small carcinoma is associated with cigarette


smoking and exposure to environmental carcinogens,
including asbestos and uranium. Chronic interstitial fibrosis
is associated with the development of adenocarcinoma of
the lung.

Front

Back

57. When admitting a patient with the diagnosis of asthma


exacerbation, the nurse will assess for which of the following
potential triggers? (Select all that apply.) A. Exercise B.
Allergies C. Emotional stress D. Decreased humidity

A,B,C Although the exact mechanism of asthma is


unknown, there are several triggers that may precipitate an
attack. These include allergens, exercise, air pollutants,
respiratory infections, drug and food additives, psychologic
factors, and GERD.

1. The arterial blood gas (ABG) readings that indicate


compensated respiratory acidosis are a PaCO2 of A. 30 mm
Hg and bicarbonate level of 24 mEq/L. B. 30 mm Hg and
bicarbonate level of 30 mEq/L. C. 50 mm Hg and bicarbonate
level of 20 mEq/L. D. 50 mm Hg and bicarbonate level of 30
mEq/L.

D. 50 mm Hg and bicarbonate level of 30 mEq/L. If


compensation is present, carbon dioxide and bicarbonate
are abnormal (or nearly so) in opposite directions (e.g., one
is acidotic and the other alkalotic).

2. A patient admitted to the emergency department with


tension pneumothorax and mediastinal shift following an
automobile crash is most likely to exhibit A. bradycardia. B.
severe hypotension. C. mediastinal flutter. D. a sucking chest
wound.

B. severe hypotension. Mediastinal shift may cause


compression of the lung in the direction of the shift and
compression, traction, torsion, or kinking of the great
vessels. Blood return to the heart is dangerously impaired
and causes a subsequent decrease in cardiac output and
blood pressure. Tachycardia is a clinical manifestation of
tension pneumothorax. An uncovered opened
pneumothorax is associated with a sucking chest wound
and mediastinal flutter.

3. In preparing the preoperative teaching plan for a patient


who is to undergo a total laryngectomy, a nurse should give
highest priority to the A. tracheostomy being in place for 2 to
3 days. B. patient's not being able to speak normally again.
C. insertion of a gastrostomy feeding tube during surgery. D.

B. patient's not being able to speak normally again. Patients


who have a total laryngectomy have a permanent
tracheostomy and will need to learn how to speak using
alternative methods, such as an artificial larynx. The
tracheostomy will be permanent to allow normal breathing

Front

Back

patient's not being able to perform deep-breathing exercises.

patterns and air exchange. After surgery, the patient's


nutrition is supplemented with enteral feedings, and when
the patient can swallow secretions, oral feedings can begin.
Deep-breathing exercises should be performed with the
patient at least every 2 hours to prevent further pulmonary
complications.

4. After a posterior nasal pack is inserted by a physician, the


patient is very anxious and states, "I don't feel like I'm
breathing right." The immediate intervention the nurse should
initiate is to A. monitor ABGs. B. reassure the patient that this
is normal discomfort. C. cut the pack strings and pull the
packing out with a hemostat. D. direct a flashlight into the
patient's mouth and inspect the oral cavity.

D. direct a flashlight into the patient's mouth and inspect the


oral cavity. The nurse should inspect the oral cavity for the
presence of blood, soft palate necrosis, and proper
placement of the posterior plug. If the posterior plug is
visible, the physician should be notified for readjustment of
the packing. Reassurance, cutting the strings, and ABGs
are not top priority interventions. The nurse needs further
data before intervening.

5. A nurse is performing assessment for a patient diagnosed


with chronic obstructive pulmonary disease (COPD). Which
of the following findings should the nurse expect to observe?
A. Nonproductive cough B. Prolonged inspiration C.
Vesicular breath sounds D. Increased anterior-posterior
chest diameter

D. Increased anterior-posterior chest diameter An


increased anterior-posterior diameter is a compensatory
mechanism experienced by patients with COPD and is
caused by air-trapping. Patients with COPD have a
productive cough, often expectorating copious amounts of
sputum. Because of air-trapping, patients with COPD
experience a prolonged expiration because the rate of gas
on exhalation takes longer to escape. Chest auscultation
for patients with COPD often reveals wheezing, crackles,
and other adventitious breath sounds.

6. A nurse is working on a respiratory care unit where many

A. Wearing perfume to work People with asthma should

Front

Back

of the patients are affected by asthma. Which of the following


actions by the nurse would most likely increase respiratory
difficulty for the patients? A. Wearing perfume to work B.
Encouraging patients to ambulate daily C. Allowing the
patients to eat green leafy vegetables D. Withholding
antibiotic therapy until cultures are obtained

avoid extrinsic allergens and irritants (e.g., dust, pollen,


smoke, certain foods, colognes and perfumes, certain types
of medications) because their airways become inflamed,
producing shortness of breath, chest tightness, and
wheezing. Many green leafy vegetables are rich in vitamins,
minerals, and proteins, which incorporate healthy lifestyle
patterns into the patients' daily living routines. Routine
exercise is a part of a prudent lifestyle, and for patients with
asthma the physical and psychosocial effects of ambulation
can incorporate feelings of well-being, strength, and
enhancement of physical endurance. Antibiotic therapy is
always initiated after cultures are obtained so that the
sensitivity to the organism can be readily identified.

7. The most appropriate position to assist a patient with


chronic obstructive pulmonary disease (COPD) who is having
difficulty breathing would be a A. high Fowler's position
without a pillow behind the head. B. semi-Fowler's position
with a single pillow behind the head. C. right side-lying
position with the head of the bed at 45 degrees' elevation. D.
sitting upright and forward position with arms supported on
an over-the-bed table.

D. sitting upright and forward position with arms supported


on an over-the-bed table.Sitting upright and leaning forward
with arms supported on an over-the-bed table would be of
most help to this patient, because it allows for expansion of
the thoracic cage in all four directions (front, back, and two
sides).

8. A person complains of fatigue and malaise and has a


slight temperature elevation for 2 days before symptoms of
influenza (fever, chest congestion, and productive cough)
become noticeable. During the time immediately before the
illness is diagnosed, the patient A. could avoid contracting

C. is in the prodromal stage and is highly contagious and


able to spread the disease. The prodromal stage is a short
period of time (hours to several days) immediately
preceding the onset of an illness during which the patient is
very contagious. Antibiotics are not effective against viral

Front

Back

the disease if treatment is begun with antibiotics. B. is unable


to spread the disease because it is still in the incubation
period. C. is in the prodromal stage and is highly contagious
and able to spread the disease. D. has a nosocomial
infection, which affects approximately two million individuals
a year.

illnesses. The incubation period is the time from entry of the


organism to the onset of symptoms and, in some viral
illnesses, may be contagious. Nosocomial infections are
those acquired in a hospital, and this scenario does not
suggest the source of the infection.

9. In older adults, infection after exposure to respiratory


illness is most likely to A. result in similar rates of infection as
in the younger adult. B. be easily prevented with the use of
antibiotics after being exposed. C. result in serious lower
respiratory infection related to weakened respiratory muscles
and fewer cilia. D. be less serious because the older adult
has less contact with younger children who are most likely to
carry serious infections.

C. result in serious lower respiratory infection related to


weakened respiratory muscles and fewer cilia. Changes in
the older adult respiratory system make older adults more
susceptible to infections that can be very serious and life
threatening. Use of antibiotics to "prevent" lung infections is
not recommended and is ineffective for viral infections.

10. If a nurse is caring for an 80-year-old patient with a


temperature of 100.4 F, crackles at the right lung base, pain
with deep inspiration, and dyspnea, which of the following
orders is the nurse's priority? A. Sputum specimen for culture
and sensitivity B. Codeine 15 mg orally every 6 hours as
needed C. Incentive spirometer every 2 hours while awake
D. Amoxicillin (Amoxil) 500 mg orally 4 times a day

A. Sputum specimen for culture and sensitivity The patient


presents with signs of a respiratory infection. To initiate the
most effective therapy, the health care prescriber must
know the pathogen causing the infection. Therefore, the
sputum specimen is the nurse's priority. If the antibiotic is
administered before the specimen is obtained, the results of
the culture might not be as accurate and could impair the
effectiveness of therapy. After the specimen is obtained,
the nurse can administer codeine for coughing and begin
the incentive spirometry to mobilize secretions and improve
the patient's ability to expectorate the secretions.

Front

Back

27. A patient with COPD is receiving oxygen at 2 L/min.


While in the supine position for a bath, the patient complains
of shortness of breath. What is the most appropriate first
nursing action? A. Increase the flow of oxygen. B. Perform
tracheal suctioning. C. Report this to the physician. D. Assist
the patient to Fowler's position.

D. Assist the patient to Fowler's position. Breathing is


easier in Fowler's position because it permits greater
expansion of the chest cavity. If repositioning does not
improve the situation, then oxygenation and physician
reporting might be appropriate. The patient would not
benefit from tracheal suctioning.

11. When assessing a patient's respiratory status, which of


the following nonrespiratory data are most important for the
nurse to obtain? A. Height and weight B. Neck circumference
C. Occupation and hobbies D. Usual daily fluid intake

C. Occupation and hobbiesMany respiratory problems


occur as a result of chronic exposure to inhalation irritants.
Common occupational sources of inhalation irritants include
mines, granaries, farms, lawn care companies, paint,
plastics and rubber manufacture, and building remodeling.
Hobbies associated with inhalation irritants include
woodworking, metal finishing, furniture refinishing, painting,
and ceramics. Daily fluids, height, and weight are more
related to respiratory problems secondary to cardiac issues.

12. If a nurse is assessing a patient whose recent blood gas


determination indicated a pH of 7.32 and respirations are
measured at 32 breaths/min, which of the following is the
most appropriate nursing assessment? A. The rapid
breathing is causing the low pH. B. The nurse should sedate
the patient to slow down respirations. C. The rapid breathing
is an attempt to compensate for the low pH. D. The nurse
should give the patient a paper bag to breathe into to correct
the low pH.

C. The rapid breathing is an attempt to compensate for the


low pH. The respiratory system influences pH (acidity)
through control of carbon dioxide exhalation. Thus, rapid
breathing increases the pH. Breathing into a paper bag aids
a patient who is hyperventilating; in respiratory alkalosis, it
aids in lowering the pH. The use of sedation can cause
respiratory depression and hypoventilation, resulting in an
even lower pH.

13. If a patient with an uncuffed tracheostomy tube coughs

B. attempt to reinsert the tracheostomy tube.Retention

Front

Back

violently during suctioning and dislodges the tracheostomy


tube, a nurse should first A. call the physician. B. attempt to
reinsert the tracheostomy tube. C. position the patient in a
lateral position with the neck extended. D. cover the stoma
with a sterile dressing and ventilate the patient with a manual
bag-mask until the physician arrives.

sutures may be grasped (if present) and the tracheostomy


opening spread, or a hemostat may be used to spread the
opening. The obturator is inserted into the replacement
tube (one size smaller than the original tube), lubricated
with saline solution, and inserted into the stoma at a 45degree angle to the neck. If the attempt is successful, the
obturator tube should immediately be removed.

14. Upon entering the room of a patient who has just


returned from surgery for total laryngectomy and radical neck
dissection, a nurse should recognize a need for intervention
when finding A. a gastrostomy tube that is clamped. B. the
patient coughing blood-tinged secretions from the
tracheostomy. C. the patient positioned in a lateral position
with the head of the bed flat. D. 200 ml of serosanguineous
drainage in the patient's portable drainage device.

C. the patient positioned in a lateral position with the head


of the bed flat. After total laryngectomy and radical neck
dissection, a patient should be placed in a semi-Fowler's
position to decrease edema and limit tension on the suture
line.

15. When administering oxygen to a patient with COPD with


the potential for carbon dioxide narcosis, the nurse should A.
never administer oxygen at a rate of more than 2 L/min. B.
monitor the patients use of oxygen to detect oxygen
dependency. C. monitor the patient for symptoms of oxygen
toxicity, such as paresthesias. D. use ABGs as a guide to
determine what FIO2 level meets the patients needs.

D. use ABGs as a guide to determine what FIO2 level


meets the patients needs. It is critical to start oxygen at low
flow rates and then use ABGs as a guide to determine what
FIO2 level is sufficient and can be tolerated.

16. To ensure the correct amount of oxygen delivery for a


patient receiving 35% oxygen via a Venturi mask, it is most
important that the nurse A. keep the air-entrainment ports

A. keep the air-entrainment ports clean and unobstructed.


Oxygen is delivered to a small jet in the center of a widebased cone. Air is entrained (pulled through) openings in

Front

Back

clean and unobstructed. B. apply an adaptor to increase


humidification of the oxygen. C. drain moisture condensation
from the oxygen tubing every hour. D. keep the flow rate high
enough to keep the bag from collapsing during inspiration.

the cone as oxygen flows through the small jet. The degree
of restriction or narrowness of the jet determines the
amount of entrainment and the dilution of pure oxygen with
room air and thus the concentration of oxygen. Although
applying an adaptor can increase the humidification with
the Venturi mask, it is not the best answer, because an
open port is essential to proper functioning. Draining
moisture condensation from the oxygen tubing is performed
as often as needed, not on an hourly schedule. A plastic
face mask with a reservoir bag needs to have sufficient flow
rate to keep the bag inflated.

17. While caring for a patient with respiratory disease, a


nurse observes that the oxygen saturation drops from 94% to
85% when the patient ambulates. The nurse should
determine that A. supplemental oxygen should be used when
the patient exercises. B. ABG determinations should be done
to verify the oxygen saturation reading. C. this finding is a
normal response to activity and that the patient should
continue to be monitored. D. the oximetry probe should be
moved from the finger to the earlobe for an accurate oxygen
saturation measurement during activity.

A. supplemental oxygen should be used when the patient


exercises.An oxygen saturation lower than 90% indicates
inadequate oxygenation. If the drop is related to activity of
some type, supplemental oxygen is indicated.

18. A nurse establishes the presence of a tension


pneumothorax when assessment findings reveal a(n) A.
absence of lung sounds on the affected side. B. inability to
auscultate tracheal breath sounds. C. deviation of the
trachea toward the side opposite the pneumothorax. D. shift

C. deviation of the trachea toward the side opposite the


pneumothorax. Tension pneumothorax is caused by rapid
accumulation of air in the pleural space, causing severely
high intrapleural pressure. This results in collapse of the
lung, and the mediastinum shifts toward the unaffected

Front

Back

of the point of maximal impulse (PMI) to the left, with


bounding pulses.

side, which is subsequently compressed.

28. To find the infection site associated with acute


lymphangitis, the nurse should look _____ to the
inflammation. A. distal B. anterior C. proximal D. contralateral

A. distal The nurse should assess distal to swelling to


locate the initial site of infection. Examining proximal,
contralateral, or anterior to the inflammation does not
describe swelling associated with infection.

19. Which of the following statements made by a nurse would


indicate proper teaching principles regarding feeding and
tracheostomies? A. Follow each spoon of food consumed
with a drink of fluid. B. Thin your foods to a liquid
consistency whenever possible. C. Tilt your chin forward
toward the chest when swallowing your food. D. Make sure
your cuff is overinflated before eating if you have swallowing
problems.

C. Tilt your chin forward toward the chest when swallowing


your food. A nurse should instruct a patient to tilt the chin
toward the chest, which will close the glottis and allow food
to enter the normal passageway. Ideally, foods should be of
a thick consistency to enable effective swallowing and
reduce the risk of aspiration. Overinflation of the cuff
causes swallowing difficulties. Fluids should be consumed
in small amounts after swallowing to prevent the risk of
aspiration.

20. If a patient states, "It's hard for me to breathe and I feel


short-winded all the time," what is the most appropriate
terminology to be applied in documenting this assessment by
a nurse? A. Apnea B. Dyspnea C. Tachypnea D. Respiratory
fatigue

B. Dyspnea Dyspnea is a subjective description reflective of


the patient's statement indicating difficulty in breathing.
Apnea refers to absence of breath or breathing. Tachypnea
refers to an increased rate of breathing, usually greater
than 20 breaths per minute. Respiratory fatigue is
subjective and usually refers to the patient exhibiting signs
and symptoms associated with a comprehensive
respiratory assessment including laborious breathing, use
of accessory muscles, and slowing of respirations.

Front

Back

21. To prevent atelectasis in an 82-year-old patient with a hip


fracture, a nurse should A. supply oxygen. B. suction the
upper airway. C. ambulate the patient frequently. D. assist
the patient with aggressive coughing and deep breathing.

D. assist the patient with aggressive coughing and deep


breathing. Decreased mobility after surgery in older adults
creates the possibility of fluid buildup and retention in lung
tissue. One of the primary goals of nursing intervention is to
prevent atelectasis in a high-risk patient. Aggressive
coughing and deep breathing can prevent atelectasis in the
postoperative patient.

22. Respiratory acidosis is at highest risk in a patient with A.


hypokalemia. B. pulmonary fibrosis. C. salicylate overdose.
D. COPD.

D. COPD. Chronic respiratory acidosis is most commonly


caused by COPD. Pulmonary fibrosis, hypokalemia, and
salicylate overdose do not predispose a patient to
respiratory acidosis. Hypokalemia can lead to cardiac
dysrhythmias. Salicylate overdose results in central
nervous system changes, and pulmonary fibrosis can result
in respiratory arrest.

40. Select all that apply. Atelectasis can be caused by A.


long-term smoking. B. inadequate surfactant. C. localized
airway obstruction. D. an increase in lung expansion. E. an
increase in elastic recoil.

BCE The collapse of lung tissue has several causes,


including reduced lung expansion, localized airway
obstruction, inadequate surfactant, and an increase in
elastic recoil. Smoking, although harmful, does not in itself
cause atelectasis.

23. A patient is having inspiratory stridor (crowing respiration)


and the nurse suspects he is experiencing a laryngospasm.
Which of the following would be most appropriate to
implement for a patient experiencing a laryngospasm? A.
Administer 100% oxygen. B. Position the patient in high
Fowler's position. C. Insert a 16-gauge (large-bore) IV

A. Administer 100% oxygen.A nurse should immediately


administer 100% oxygen to the patient until the airway is
fully reestablished, the larynx relaxes, and the spasms stop.
Activating the emergency response team is not an
immediate nursing action at this time because the nurse
can administer the oxygen without the assistance of others.

Front

Back

needle. D. Activate the emergency response team (code blue


team) to the patient's room.

Positioning the patient in high Fowler's will not address the


patient's need for immediate reoxygenation because of the
patient's compromised respiratory state. Insertion of an IV
device is not the first priority response but should be
implemented after the nurse has assessed that the airway
is stable.

24. A nurse is preparing to establish oxygen therapy for a


patient with COPD, and the physician's prescription reads
"oxygen per nasal cannula at 5 L per minute." Which of the
following actions should the nurse take? A. Administer the
oxygen as prescribed. B. Call the physician and question the
correct flow rate of the oxygen. C. Establish the oxygen as
prescribed and obtain an ABG. D. Change the delivery
device from a nasal cannula to a simple oxygen mask.

B. Call the physician and question the correct flow rate of


the oxygen. The nurse should call the physician
immediately and question the flow rate for delivery of the
oxygen before implementation. Oxygen is used cautiously
in patients with COPD because of longstanding hypoxemia
serving as the respiratory drive mechanism. If high levels of
oxygen are administered, the respiratory drive can be
obliterated. Changing the device to a simple oxygen mask
may alter the oxygen concentration being delivered to the
patient and will further enhance the obliteration of the
patient's respiratory drive. Obtaining an ABG sample is not
a priority at this time, and the action does not address the
validity of the prescribed oxygen dosing for the patient.

25. A 75-year-old obese patient who is snoring loudly and


having periods of apnea several times each night is most
likely experiencing A. narcolepsy. B. sleep apnea. C. sleep
deprivation. D. paroxysmal nocturnal dyspnea.

B. sleep apnea. Sleep apnea is most common in obese


patients. Typical symptoms include snoring and periods of
apnea. Narcolepsy is when a patient falls asleep
unexpectedly. Sleep deprivation could result from sleep
apnea. Paroxysmal nocturnal dyspnea occurs when a
patient has shortness of breath during the night.

Front

Back

26. Which of the following conditions is manifested by


unexplained shortness of breath and a high mortality rate? A.
Bleeding ulcer B. Transient ischemia C. Pulmonary embolism
D. MI

C. Pulmonary embolism A high mortality rate is associated


with a pulmonary embolism. A pulmonary embolism is an
obstruction of the pulmonary artery caused by an embolus.
It presents with hypoxia, anxiety, restlessness, and
shortness of breath. Bleeding ulcers, MI, and transient
ischemia are not associated with such a high mortality rate.

10. Anticoagulant therapy is used in the treatment of


thromboembolic disease because anticoagulants can A.
dissolve the thrombi. B. decrease blood viscosity. C. prevent
absorption of vitamin K. D. inhibit the synthesis of clotting
factors.

D. inhibit the synthesis of clotting factors. Anticoagulant


therapy is based on the premise that the initiation or
extension of thrombi can be prevented by inhibiting the
synthesis of clotting factors or by accelerating their
inactivation. The anticoagulants heparin and warfarin do not
induce thrombolysis but effectively prevent clot extension.

29. Which of the following instructions are most appropriate


in the home management of a patient who has undergone
surgery for oral cancer? A. You should drink plenty of fluids
and eat foods you enjoy. B. It is normal to have some
leakage of saliva from the suture line. C. Lying in a prone
position helps decrease swelling at the suture line. D. You
should avoid foods high in protein while your suture line is
healing.

A. You should drink plenty of fluids and eat foods you


enjoy. For patients who have undergone treatment for
head and neck cancers, maintaining adequate nutrition is a
challenge. The nurse encourages the patient to increase
fluids to prevent dehydration and liquefy secretions. These
patients are more likely to eat foods that they enjoy and can
tolerate.

30. Which of the following conditions or factors in a 64-yearold patient diagnosed with head and neck cancer most likely
contributed to this health problem? A. Patient's hobby is oil
painting. B. Patient's father also had head and neck cancer.
C. Patient uses chewing tobacco and drinks beer daily. D.

C. Patient uses chewing tobacco and drinks beer daily.


Many environmental risk factors contribute to the
development of head and neck cancer, although the actual
cause is unknown. There does not appear to be a genetic
predisposition to this type of cancer. The two most

Front

Back

Patient quit school at age 16 and has worked in a butcher


shop for more than 40 years.

important risk factors are tobacco and alcohol use,


especially in combination. Other risk factors include
chewing tobacco, pipe smoking, marijuana use, voice
abuse, chronic laryngitis, exposure to industrial chemicals
or hardwood dust, and poor oral hygiene.

31. A patient's ABGs include a PaO2 of 88 mm Hg and a


PaCO2 of 38 mm Hg and mixed venous blood gases include
a PvO2 of 40 mm Hg and PvCO2 of 46 mm Hg. These
findings indicate that the patient has A. impaired cardiac
output. B. unstable hemodynamics. C. inadequate delivery of
oxygen to the tissues. D. normal capillary oxygencarbon
dioxide exchange.

D. normal capillary oxygencarbon dioxide exchange.


Normal venous blood gas values reflect the normal uptake
of oxygen from arterial blood and the release of carbon
dioxide from cells into the blood, resulting in a much lower
PaO2 and an increased PaCO2. The pH is also decreased
in mixed venous blood gases because of the higher
PvCO2. Normal mixed venous blood gases also have much
lower PvO2 and SvO2 than arterial blood bases. Mixed
venous blood gases are used when patients are
hemodynamically unstable to evaluate the amount of
oxygen delivered to the tissue and the amount of oxygen
consumed by the tissues.

32. An excess of carbon dioxide in the blood causes an


increased respiratory rate and volume because CO2 A.
displaces oxygen on hemoglobin, leading to a decreased
PaO2. B. causes an increase in the amount of hydrogen ions
available in the body. C. combines with water to form
carbonic acid, lowering the pH of cerebrospinal fluid. D.
directly stimulates chemoreceptors in the medulla to increase
respiratory rate and volume.

C. combines with water to form carbonic acid, lowering the


pH of cerebrospinal fluid. A combination of excess CO2 and
H2O results in carbonic acid, which lowers the pH of the
cerebrospinal fluid and stimulates an increase in the
respiratory rate. Peripheral chemoreceptors in the carotid
and aortic bodies also respond to increases in PaCO2 to
stimulate the respiratory center. Excess CO2 does not
increase the amount of hydrogen ions available in the body
but does combine with the hydrogen of water to form an

Front

Back

acid.

33. A patient with an acute pharyngitis is seen at the clinic


with fever and severe throat pain that affects swallowing. On
inspection the throat is reddened and edematous with patchy
yellow exudates. The nurse anticipates that collaborative
management will include A. treatment with antibiotics. B.
treatment with antifungal agents. C. a throat culture or rapid
strep antigen test. D. treatment with medication only if the
pharyngitis does not resolve in 3 to 4 days.

C. a throat culture or rapid strep antigen test. Although


inadequately treated -hemolytic streptococcal infections
may lead to rheumatic heart disease or glomerulonephritis,
antibiotic treatment is not recommended until strep
infections are definitely diagnosed with culture or antigen
tests. The manifestations of viral and bacterial infections
are similar, and appearance is not diagnostic except when
candidiasis is present.

34. Following a supraglottic laryngectomy, the patient is


taught how to use the supraglottic swallow to minimize the
risk of aspiration. In teaching the patient about this technique,
the nurse instructs the patient to A. perform Valsalva
maneuver immediately after swallowing. B. breathe between
each Valsalva maneuver and cough sequence. C. cough
after swallowing to remove food from the top of the vocal
cords. D. practice swallowing thin, watery fluids before
attempting to swallow solid foods.

C. cough after swallowing to remove food from the top of


the vocal cords. A supraglottic laryngectomy involves
removal of the epiglottis and false vocal cords, and the
removal of the epiglottis allows food to enter the trachea.
Supraglottic swallowing requires performance of the
Valsalva maneuver before placing food in the mouth and
swallowing. The patient then coughs to remove food from
the top of the vocal cords, swallows again, and then
breathes after the food has been removed from the vocal
cords.

35. A patient is admitted to the hospital with fever, chills, a


productive cough with rusty sputum, and pleuritic chest pain.
Pneumococcal pneumonia is suspected. An appropriate
nursing diagnosis for the patient based on the patient's
manifestations is A. hyperthermia related to acute infectious
process. B. chronic pain related to ineffective pain

A. hyperthermia related to acute infectious process. The


patient with pneumococcal pneumonia is acutely ill with
fever and the systemic manifestations of fever, such as
chills, thirst, headache, and malaise. Interventions that
monitor temperature and aid in lowering body temperature
are appropriate. Ineffective airway clearance would be

Front

Back

management. C. risk for injury related to disorientation and


confusion. D. ineffective airway clearance related to retained
secretions.

manifested by adventitious breath sounds and difficulty


producing secretions. Disorientation and confusion are not
noted in this patient and are not typical unless the patient is
very hypoxemic. Pleuritic pain is an acute pain that is due
to inflammation of the pleura.

11. If a patient has pernicious anemia, the nurse should


provide information regarding A. frequent bouts of dyspnea.
B. risks relative to dehydration. C. deficiency of intrinsic
factor. D. lack of any effective treatment for this condition.

C. deficiency of intrinsic factor. Pernicious anemia is a type


of anemia caused by failure of absorption of vitamin B12
(cobalamin). The most common cause is lack of intrinsic
factor, a glucoprotein produced by the parietal cells of the
gastric lining.

36. The resurgence in TB resulting from the emergence of


multidrug-resistant strains of Mycobacterium tuberculosis is
primarily the result of A. a lack of effective means to
diagnose TB. B. poor compliance with drug therapy in
patients with TB. C. the increased population of
immunosuppressed individuals with AIDS. D. indiscriminate
use of antitubercular drugs in treatment of other infections.

B. poor compliance with drug therapy in patients with TB.


Drug-resistant strains of TB have developed because TB
patients' compliance to drug therapy has been poor and
there has been general decreased vigilance in monitoring
and follow-up of TB treatment. Antitubercular drugs are
almost exclusively used for TB infections. TB can be
effectively diagnosed with sputum cultures. The incidence
of TB is at epidemic proportions in patients with HIV, but
this does not account for drug-resistant strains of TB.

37. The chronic inflammation of the bronchi characteristic of


chronic obstructive pulmonary disease (COPD) results in A.
collapse of small bronchioles on expiration. B. permanent,
abnormal dilation of the bronchi. C. hyperplasia of mucussecreting cells and bronchial edema. D. destruction of the

C. hyperplasia of mucus-secreting cells and bronchial


edema. Chronic bronchitis is characterized by chronic
inflammation of the bronchial lining, with edema and
increased mucus production. Collapse of small bronchioles
on expiration is common in emphysema, and abnormal
dilation of the bronchi because of destruction of the elastic

Front

Back

elastic and muscular structures of the bronchial wall.

and muscular structures is characteristic of bronchiectasis.

38. In teaching the patient with COPD about the need for
physical exercise, the nurse informs the patient that A. all
patients with COPD should be able to increase walking
gradually up to 20 min/day. B. a bronchodilator inhaler should
be used to relieve exercise-induced dyspnea immediately
after exercise. C. shortness of breath is expected during
exercise but should return to baseline within 5 minutes after
the exercise. D. monitoring the heart rate before and after
exercise is the best way to determine how much exercise can
be tolerated.

C. shortness of breath is expected during exercise but


should return to baseline within 5 minutes after the
exercise.Shortness of breath usually increases during
exercise, but the activity is not being overdone if breathing
returns to baseline within 5 minutes after stopping.
Bronchodilators can be administered 10 minutes before
exercise but should not be administered for at least 5
minutes after activity to allow recovery. Patients are
encouraged to walk 15 to 20 minutes a day with gradual
increases, but actual patterns will depend on patient
tolerance. Dyspnea most frequently limits exercise and is a
better indication of exercise tolerance than is heart rate in
the patient with COPD.

39. Select all that apply. Which of the following are clinical
manifestations of tension pneumothorax? A. Midline trachea
B. Severe hypertension C. Progressive cyanosis D. A loud
bruit on affected side E. Asymmetrical chest wall movement
F. Subcutaneous emphysema in the neck

C,E, F The indicators of tension pneumothorax are


asymmetrical chest wall movement, severe hypotension,
subcutaneous emphysema in the neck and upper chest,
and progressive cyanosis.

41. Select all that apply. During initial assessment, a nurse


should record which of the following manifestations of
respiratory distress? A. Tachypnea B. Nasal flaring C.
Thready pulse D. Panting or grunting E. Use of intercostal
muscles F. An inspiratory-to-expiratory ratio of 1:2

AD Manifestations of respiratory distress include


tachypnea, grunting and panting on respiration, central
cyanosis, use of accessory muscles, and flaring nares.

Front

Back

42. Select all that apply. Which of the following nursing


actions can help clear tracheobronchial secretions in a
patient with cystic fibrosis? A. Postural drainage B.
Suppressing the cough C. Ensuring adequate hydration D.
Administering mucolytic aerosols E. Encouraging the patient
to lie flat F. Administering water-soluble vitamins

ACD Postural drainage, adequate hydration, and


administration of mucolytic aerosols all encourage coughing
and the clearing of secretions. A patient with cystic fibrosis
will be more comfortable sitting upright.

43. Select all that apply. Which of the following is included in


a comprehensive respiratory assessment? A. Pulse oximetry
B. Chest auscultation C. Apical radial pulse D. Nail-bed
assessment E. Evaluation of respiratory effort F. Rate and
character of respirations

ABDEF The total assessment of the respiratory system


includes pulse oximetry; auscultation; skin and nail-bed
assessment for the detection of cyanosis; and rate,
character, and degree of effort of respirations. The apical
radial pulse is a cardiac assessment.

1. When assessing a patient's nutritional-metabolic pattern


related to hematologic health, the nurse would: A. Inspect the
skin for petechiae. B. Ask the patient about joint pain. C.
Assess for vitamin C deficiency. D. Determine if the patient
can perform ADLs.

A. Inspect the skin for petechiae. Any changes in the skin's


texture or color should be explored when assessing the
patient's nutritional-metabolic pattern related to hematologic
health. The presences of petechiae or ecchymotic areas
could be indicative of hematologic deficiencies related to
poor nutritional intake or related causes.

20. Using light pressure with the index and middle fingers,
the nurse cannot palpate any of the patient's superficial
lymph nodes. The nurse A. records this finding as normal. B.
should reassess the lymph nodes using deeper pressure. C.
asks the patient about any history of any radiation therapy. D.
notifies the health care provider that x-rays of the nodes will
be necessary.

A. records this finding as normal. Superficial lymph nodes


are evaluated by light palpation, but they are not normally
palpable. It may be normal to find small (<1.0 cm), mobile,
firm, nontender nodes. Deep lymph nodes are detected
radiographically.

Front

Back

2. When assessing lab values on a patient admitted with


septicemia, the nurse would expect to find: A. Increased
platelets B. Decreased red blood cells C. Decreased
erythrocyte sedimentation rate (ESR) D. Increased bands in
the WBC differential (shift to the left)

D. Increased bands in the WBC differential (shift to the left)


When infections are severe, such as in septicemia, more
granulocytes are released from the bone marrow as a
compensatory mechanism. To meet the increased demand,
many young, immature polymorphonuclear neutrophils
(bands) are released into circulation. WBCs are usually
reported in order of maturity, with the less mature forms on
the left side of a written report. Hence, the term "shift to the
left" is used to denote an increase in the number of bands.

3. When caring for a patient with metastatic cancer, the nurse


notes a hemoglobin level of 8.7 g/dl and hematocrit of 26%.
The nurse would place highest priority on initiating
interventions that will reduce which of the following? A.
Fatigue B. Thirst C. Headache D. Abdominal pain

A. Fatigue The patient with a low hemoglobin and


hematocrit (normal values approximately 13.5% to 17% and
40% to 54%, respectively) is anemic and would be most
likely to experience fatigue. This symptom develops
because of the lowered oxygen-carrying capacity that leads
to reduced tissue oxygenation to carry out cellular
functions.

4. The nurse is caring for a patient who is to receive a


transfusion of two units of packed red blood cells. After
obtaining the first unit from the blood bank, the nurse would
ask which of the following health team members in the
nurses' station to assist in checking the unit before
administration? A. Unit secretary B. Another registered nurse
C. A physicians assistant D. A phlebotomist

B. Another registered nurseBefore hanging a transfusion,


the registered nurse must check the unit with another RN or
with a licensed practical (vocational) nurse, depending on
agency policy.

5. Before starting a transfusion of packed red blood cells for


an anemic patient, the nurse would arrange for a peer to

D. 15 As part of standard procedure, the nurse remains


with the patient for the first 15 minutes after hanging a

Front

Back

monitor his or her other assigned patients for how many


minutes when the nurse begins the transfusion? A. 60 B. 5 C.
30 D. 15

blood transfusion. Patients who are likely to have a


transfusion reaction will more often exhibit signs within the
first 15 minutes that the blood is infusing.

6. When preparing to administer an ordered blood


transfusion, the nurse selects which of the following
intravenous solutions to use when priming the blood tubing?
A. 5% dextrose in water B. Lactated Ringers C. 0.9% sodium
chloride D. 0.45% sodium chloride

C. 0.9% sodium chloride The blood set should be primed


before the transfusion with 0.9% sodium chloride, also
known as normal saline. It is also used to flush the blood
tubing after the infusion is complete to ensure the patient
receives blood that is left in the tubing when the bag is
empty.

7. The nurse notes a physician's order written at 10:00 AM


for 2 units of packed red blood cells to be administered to a
patient who is anemic secondary to chronic blood loss. If the
transfusion is picked up at 11:30, the nurse should plan to
hang the unit no later than which of the following times? A.
11:45 AM B. 12:00 noon C. 12:30 PM D. 3:30 PM

B. 12:00 noon The nurse must hang the unit of packed red
blood cells within 30 minutes of signing them out from the
blood bank

8. The nurse receives a physician's order to transfuse fresh


frozen plasma to a patient suffering from an acute blood loss.
Which of the following procedures is most appropriate for
infusing this blood product? A. Hand the fresh frozen plasma
as a piggyback to a new bag of primary IV solution without
KCl. B. Infuse the fresh frozen plasma as rapidly as the
patient will tolerate. C. Hang the fresh frozen plasma as a
piggyback to the primary IV solution. D. Infuse the fresh
frozen plasma as a piggyback to a primary solution of normal

B. Infuse the fresh frozen plasma as rapidly as the patient


will tolerate. The fresh frozen plasma should be
administered as rapidly as possible and should be used
within 2 hours of thawing. Fresh frozen plasma is infused
using any straight-line infusion set. Any existing IV should
be interrupted while the fresh frozen plasma is infused,
unless a second IV line has been started for the
transfusion.

Front

Back

saline.

9. Before beginning a transfusion of RBCs, which of the


following actions by the nurse would be of highest priority to
avoid an error during this procedure? A. Check the identifying
information on the unit of blood against the patients ID
bracelet. B. Select new primary IV tubing primed with
lactated Ringers solution to use for the transfusion. C. Add
the blood transfusion as a secondary line to the existing IV
and used the IV controller to maintain correct flow. D.
Remain with the patient for 60 minutes after beginning the
transfusion to watch for signs of a transfusion reaction. The
patient's identifying information (name, date of birth, medical
record number) on the identification bracelet should exactly
match the information on the blood bank tag that has been
placed on the unit of blood. If any information does not
match, the transfusions should not be hung because of
possible error and risk to the patient.

he patient's identifying information (name, date of birth,


medical record number) on the identification bracelet
should exactly match the information on the blood bank tag
that has been placed on the unit of blood. If any information
does not match, the transfusions should not be hung
because of possible error and risk to the patient.

10. The blood bank notifies the nurse that the two units of
blood ordered for an anemic patient are ready for pick up.
The nurse should take which of the following actions to
prevent an adverse effect during this procedure? A.
Immediately pick up both units of blood from the blood bank.
B. Regulate the flow rate so that each unit takes at least 4
hours to transfuse. C. Set up the Y-tubing of the blood set
with dextrose in water as the flush solution. D. Infuse the

D. Infuse the blood slowly for the first 15 minutes of the


transfusion. Because a transfusion reaction is more likely to
occur at the beginning of a transfusion, the nurse should
initially infuse the blood at a rate no faster than 2 ml/min
and remain with the patient for the first 15 minutes after
hanging a unit of blood.

Front

Back

blood slowly for the first 15 minutes of the transfusion.

1. A nurse is reviewing the hematologic test results for a


patient in whom the hematocrit (Hct) is reported at a reading
of 30%. Based on this result, the nurse should interpret that
the patient A. is susceptible to bleeding disorders. B. has
fewer red blood cells than normal. C. is experiencing an
inflammatory response. D. is experiencing an acute
hemolytic crisis.

B. has fewer red blood cells than normal.The Hct is the


measure of the volume of red blood cells in whole blood
expressed as a percentage. This test is useful in the
diagnosis of anemia, polycythemia, and abnormal hydration
states. Patients who are susceptible to bleeding disorders
likely will have a low platelet count. The inflammatory
response may best be evaluated by examination of results
that include the white blood cell count with differential
analysis. Acute hemolytic crisis develops in patients
receiving blood components in which incompatibility occurs
or in patients with bleeding disorders or conditions that
promote cellular damage, such as damage associated with
shock.

2. Which of the following is a factor significant in the


development of anemia in men? A. Condom use B. Large
hemorrhoids C. A diet high in cholesterol D. Smoking one
pack of cigarettes daily

B. Large hemorrhoids Gastrointestinal (GI) tract bleeding is


a common etiologic factor in men and may result from
peptic ulcers, hiatal hernia, gastritis, cancer, hemorrhoids,
diverticula, ulcerative colitis, or salicylate poisoning.

3. Following a patient's bone marrow aspiration, which of the


following nursing interventions should a nurse anticipate? A.
Application of firm pressure to the site B. Positioning the
patient in a prone position C. Positioning the patient in a
supine position D. Application of a warm, moist compress to
the site

A. Application of firm pressure to the site After a bone


marrow aspiration procedure, a nurse should apply
pressure to the aspiration site until bleeding stops.
Application of a warm, moist compress will not alter the
potential for bleeding. Positioning the patient to assume a
supine or prone position also will not address the need to

Front

Back

control bleeding from the aspiration site.

4. The nurse is teaching a patient who is to undergo bone


marrow aspiration. Which of the following statements made
by the nurse would indicate correct instruction regarding the
site for the aspiration procedure? A. The health care
provider will perform the aspiration by needle to the femur.
B. The health care provider will perform the aspiration by
needle to the scapula. C. The health care provider will
perform the aspiration by needle to the antecubital fossa. D.
The health care provider will perform the aspiration by
needle to the posterior iliac crest.

D. The health care provider will perform the aspiration by


needle to the posterior iliac crest. Bone marrow samples
are commonly taken from the posterior iliac crest or, as an
alternative, the sternum may be aspirated. These sites
provide relative ease in accessing the bone marrow via the
biopsy needle. The antecubital fossa, femur, and scapula
do not allow access to bone marrow while also providing
reduced risk of harm to the patient.

5. Absorption of vitamin B12 may be decreased in older


adults because of decreased A. intestinal motility. B.
production of bile by the liver. C. production of intrinsic factor
by the stomach. D. synthesis of cobalamin (vitamin B12) by
intestinal bacteria.

C. production of intrinsic factor by the stomach. Older


persons are at risk for deficiency of cobalamin (pernicious
anemia) because of a naturally occurring reduction of the
intrinsic factor by the stomach mucosa. Absorption of
cobalamin relies on intrinsic factor. Both must be present
for absorption. Megaloblastic anemia is related to folate
dysfunction. Intestinal motility (peristalsis) is the motion that
moves food down the GI tract. The rhythmic contractions of
muscles cause wave-like motions. Lack of peristalsis is
called "paralytic ileus." Bile is produced in the liver, is stored
and concentrated in the gallbladder, and is released into
the duodenum when fat is eaten. Bile emulsifies fats and
prepares them for enzyme digestion in order for the nutrient
to be absorbed into lymph and eventually into blood vessels
to the liver. Vitamin K (the blood-clotting vitamin) is

Front

Back

synthesized by intestinal bacteria.

6. What type of anemia is associated with folate deficiency?


A. Microcytic B. Pernicious C. Megaloblastic D. Iron
deficiency

C. Megaloblastic Megaloblastic anemia is nutritional


anemia; large immature red blood cells with a decreased
oxygen-carrying capacity can occur as a result of impaired
DNA synthesis. Folic acid is used in the synthesis of DNA
and helps convert B12 to coenzyme form. Folic acid is
needed for growth and development of red blood cells.
Microcytic anemia is anemia with abnormally small
erythrocytes (red blood cells) in Hb. This anemia is
associated with vitamin B6 (pyridoxine) deficiency.
Pernicious anemia is caused by a deficiency of vitamin
B12. Iron-deficiency anemia results from loss of blood or
deficient intake of iron foods or disease states in which the
body does not absorb or utilize iron as it should.

7. During care of a patient with multiple myeloma, an


important nursing intervention is A. limiting activity to prevent
pathologic fractures. B. assessing for changes in size and
characteristics of lymph nodes. C. maintaining a fluid intake
of 3 to 4 L/day to dilute calcium load. D. administering
narcotic analgesics continuously to control bone pain.

C. maintaining a fluid intake of 3 to 4 L/day to dilute calcium


load. Adequate hydration must be maintained to minimize
problems from hypercalcemia. The goal of a urinary output
of 1.5 to 2 L/day requires an intake of 3 to 4 L/day.

8. While obtaining the admission assessment data, which of


the following characteristics would a nurse expect a patient
with anemia to report? A. Palpitations B. Blurred vision C.
Increased appetite D. Feeling of warm flushing sensation

A. Palpitations Patients experiencing moderate anemia


(hemoglobin [Hb] 6 to 10 g/dL) may experience dyspnea
(shortness of breath), palpitations, diaphoresis (profound
perspiration) with exertion, and chronic fatigue. Blurred
vision is associated in patients experiencing profound

Front

Back

anemia states. Anorexia is common in patients with severe


anemia, as well. Patients with anemia often appear pale
and complain of feeling cold because of compensatory
vasoconstriction of the subcutaneous capillaries.

9. If a health care provider is planning to transfuse a patient


with a unit of packed red blood cells, which of the following
solutions should the health care provider hang with the
transfusion? A. 5% dextrose in water B. 0.9% sodium
chloride C. 5% dextrose in 0.9% sodium chloride D. 5%
dextrose in lactated Ringers solution

B. 0.9% sodium chloride The only solution appropriate for


administration with whole blood or blood products is 0.9%
sodium chloride. The other options are not appropriate for
use with blood products.

12. A nurse is providing care to an adult female patient and


observes that the Hb laboratory analysis result is 9 g/dl.
Based on this finding, the nurse should expect to observe A.
dyspnea. B. bradycardia. C. warm, dry skin. D. activity
tolerance without complaint of fatigue.

A. dyspnea.Hb levels are used to determine the severity of


anemia. Patients with moderate anemia (Hb 6 to 10 g/dL)
may suffer from dyspnea, palpitations, diaphoresis with
exertion, and chronic fatigue. Patients who are anemic
usually have cool skin related to compensatory mechanism
of mild vasoconstriction. Patients who are anemic
experience tachycardia because of increased demands
placed on the heart to meet overall metabolic requirements.
Activity tolerance without complaint is not correct because
patients with anemic conditions fatigue readily.

13. If a patient with arthritis develops iron-deficiency anemia,


a nurse should ask about the patient's use of A. alcoholic
beverages. B. stool softeners and laxatives. C. caffeinated
foods and beverages. D. NSAIDs.

D. NSAIDs. NSAIDs decrease the level of vitamin C, which


aids in the absorption of iron. These drugs also compete
with folate and vitamin K and may cause gastritis.
Excessive alcoholic beverage consumption can cause
stomach irritation; alcohol would not be directly related to

Front

Back

iron-deficiency anemia unless bleeding ulcers or gastritis


were to occur. NSAID consumption, not stool softeners and
laxative use, would be suspect for iron-deficiency anemia.
Caffeinated foods and beverages can cause gastric
irritation and discomfort but are not associated with irondeficiency anemia.

14. Which of the following foods is high in iron? A. Citrus


fruits B. Milk products C. Yellow vegetables D. Green leafy
vegetables

D. Green leafy vegetables Green leafy vegetables are high


in iron. Foods cooked in iron pots and foods such as liver
(the richest source), oysters, lean meats, kidney beans,
whole wheat bread, kale, spinach, egg yolk, turnip tops,
beet greens, carrots, apricots, and raisins are also high in
iron.

15. When reviewing the results of a 83-year-old patient's


blood tests, which of the following findings would be of most
concern to the nurse? A. Platelets of 150,000/l B. Serum
iron of 50 mcg/dl C. Partial thromboplastin time (PTT) of 60
seconds D. Erythrocyte sedimentation rate (ESR) of 35 mm
in 1 hour

C. Partial thromboplastin time (PTT) of 60 seconds In


aging, the partial thromboplastin time (PTT) is normally
decreased, so an abnormally high PTT of 60 seconds is an
indication that bleeding could readily occur. Platelets are
unaffected by aging, and 150,000 is a normal count. Serum
iron levels are decreased and the erythrocyte
sedimentation rate (ESR) is significantly increased with
aging, as are reflected in these values.

16. If a patient with blood type O Rh is given AB Rh blood,


the nurse would expect A. the patients Rh factor to react with
the RBCs of the donor blood. B. no adverse reaction
because the patient has no antibodies against the donor
blood. C. the anti-A and anti-B antibodies in the patients

C. the anti-A and anti-B antibodies in the patients blood to


hemolyze the donor blood. A patient with O Rh+ blood has
no A or B antigens on the red cell but does have anti-A and
anti-B antibodies in the blood and has an Rh antigen. AB
Rh blood has both A and B antigens on the red cell but no

Front

Back

blood to hemolyze the donor blood. D. the anti-A and anti-B


antibodies in the donor blood to hemolyze the patients blood.

Rh antigen and no anti-A or anti-B antibodies. If the AB Rh


blood is given to the patient with O Rh+ blood, the
antibodies in the patient's blood will react with the antigens
in the donor blood, causing hemolysis of the donor cells.
There will be no Rh reaction because the donor blood has
no Rh antigen.

17. The nurse evaluates that teaching for the patient with iron
deficiency anemia has been effective when the patient states
A. "I will need to take the iron supplements the rest of my
life." B. "I will increase my dietary intake of milk and milk
products." C. "I should increase my activity to increase my
aerobic capacity." D. "I should take the iron for several
months after my blood is normal."

D. "I should take the iron for several months after my blood
is normal." To replace the body's iron stores, iron
supplements should be continued for 2 to 3 months after
the Hb level returns to normal, but if the cause of the iron
deficiency is corrected, the supplements do not need to be
taken for a lifetime. Milk and milk products are poor sources
of dietary iron. Activity should be gradually increased as Hb
levels return to normal because aerobic capacity can be
increased when adequate Hb is available.

18. Nursing interventions for the patient with aplastic anemia


are directed toward the prevention of the complications of A.
fatigue and dyspnea. B. hemorrhage and infection. C.
thromboemboli and gangrene. D. cardiac arrhythmias and
heart failure.

B. hemorrhage and infection. Hemorrhage from


thrombocytopenia and infection from neutropenia are the
greatest risks for the patient with aplastic anemia. The
patient will experience fatigue from anemia, but bleeding
and infection are the major causes of death in aplastic
anemia.

19. A patient with hemophilia is hospitalized with acute knee


pain and swelling. An appropriate nursing intervention for the
patient includes A. wrapping the knee with an elastic
bandage. B. placing the patient on bed rest and applying ice

B. placing the patient on bed rest and applying ice to the


joint. During an acute bleeding episode in a joint, it is
important to totally rest the involved joint and slow bleeding
with application of ice. Drugs that decrease platelet

Front

Back

to the joint. C. gently performing range-of-motion (ROM)


exercises to the knee to prevent adhesions. D. administering
nonsteroidal anti-inflammatory drugs (NSAIDs) as needed for
pain.

aggregation, such as aspirin or NSAIDs, should not be


used for pain. As soon as bleeding stops, mobilization of
the affected area is encouraged with range-of-motion
(ROM) exercises and physical therapy.

21. Select all that apply. Which of the following are significant
risk factors for leukemia? A. Being a longtime smoker B.
Employment in an oil refinery C. History of hemophilia in
parent D. Having Down syndrome E. Having a twin brother
with leukemia F. Treatment with an alkylating agent = 3 years
ago

BDEF Exposure to chemical agents, treatment with


alkylating cancer drugs, leukemia in a sibling, and the
patient's having Down syndrome are all risk factors for
leukemia.

absorption atelectasis

alveolar collapse that occurs when high concentrations of


oxygen are given and oxygen replaces nitrogen in the
alveoli; if airway obstruction occurs, the oxygen is absorbed
into the bloodstream and the alveoli collapse.

acute bronchitis

an inflammation of the lower respiratory tract that is usually


due to infection.

adventitious sounds

extra breath sounds that are not normally heard, such as


crackles, rhonchi, wheezes, and pleural friction rubs.

allergic rhinitis

the reaction of the nasal mucosa to a specific allergen.

Front

Back

?-antitrypsin

a serum protein produced by the liver normally found in the


lungs that inhibits proteolytic enzymes of white cells from
lysing lung tissue; genetic deficiency of this protein can
cause emphysema.

apnea

an absence of spontaneous respirations.

asthma

a chronic inflammatory lung disease that results in airflow


obstruction; characterized by recurring episodes of
paroxysmal dyspnea, wheezing on expiration and/or
inspiration caused by constriction of the bronchi, coughing,
and viscous mucoid bronchial secretions.

centrilobular emphysema

type of emphysema often associated with chronic bronchitis


in which respiratory bronchioles enlarge, the walls are
destroyed, and the bronchioles become confluent;
characterized by enlargement of air spaces in the proximal
part of the acinus, primarily at the level of the respiratory
bronchioles.

chemoreceptor

a sensory nerve cell that responds to a change in the


chemical composition (PaCO2 and pH) of the fluid around
it.

chest percussion

rhythmic percussion of a patient's chest with cupped hands

Front

Back

to loosen retained respiratory secretions.

chest physiotherapy

a series of maneuvers including percussion, vibration, and


postural drainage designed to promote clearance of
excessive respiratory secretions.

chronic bronchitis

obstructive pulmonary disease characterized by excessive


production of mucus and chronic inflammatory changes in
the bronchi, resulting in a cough with expectoration for at
least 3 months of the year for more than 2 consecutive
years.

chronic pancreatitis

pulmonary disease state characterized by the presence of


airflow obstruction caused by chronic bronchitis or
emphysema; clinical use of the term indicates the presence
of chronic bronchitis and/or emphysema; includes asthma,
chronic bronchiectasis, chronic bronchitis, and emphysema.

chylothorax

a condition marked by lymphatic fluid in the pleural space


caused by a leak in the thoracic duct.

community-acquired pneumonia

a lower respiratory tract infection of the lung parenchyma


with onset in the community or during the first 2 days of
hospitalization.

Front

Back

compliance

a measure of the ease of expansion of the lungs and


thorax.

cor pulmonale

hypertrophy of the right side of the heart, with or without


heart failure, resulting from pulmonary hypertension.

crackle

short, low-pitched sounds consisting of discontinuous


bubbling caused by air passing through airway
intermittently occluded by mucus, unstable bronchial wall,
or fold of mucosa; evident on inspiration and, at times,
expiration; similar sound to blowing through a straw under
water.

cystic fibrosis

an autosomal recessive, multisystem disease characterized


by altered function of the exocrine glands involving primarily
the lungs, pancreas, and sweat glands.

deviated septum

a deflection of the normally straight nasal septum.

diaphragmatic breathing

breathing with the use of the diaphragm to achieve


maximum inhalation and slow respiratory rate.

dry powder inhaler

dry powdered drug delivered by inhalation.

Front

Back

dyspnea

shortness of breath; difficulty breathing that may be caused


by certain heart conditions, strenuous exercise, or anxiety.

elastic recoil

the tendency for the lungs to recoil or reduce in volume


after being stretched or expanded.

emphysema

an abnormal condition of the pulmonary system,


characterized by overinflation and destructive changes in
alveolar walls.

empyema

an accumulation of purulent exudates in a body cavity,


especially the pleural space, as a result of bacterial
infection, such as pleurisy or tuberculosis.

epistaxis

nosebleed.

esophageal speech

a method of swallowing air, trapping it in the esophagus,


and releasing it to create sound.

flail chest

instability of the chest wall resulting from multiple rib


fractures.

fremitus

vibration of the chest wall produced by vocalization.

Front

Back

hemothorax

accumulation of blood in the pleural space.

hospital-acquired pneumonia

pneumonia occurring 48 hours or longer after hospital


admission and not incubating at the time of hospitalization.

hypercapnia

greater than normal amounts of carbon dioxide in the blood


(PaCO2 > 45 mm Hg); also called hypercarbia.

hyperreactivity

an abnormal condition in which responses to stimuli are


exaggerated.

hyperresponsiveness

excessive or exaggerated response to a stimulus; in


asthma leads to bronchoconstriction in response to
physical, chemical, or pharmacologic stimuli.

hypocapnia

low arterial carbon dioxide pressure (PaCO2 < 35 mm Hg);


also called hypocarbia.

lung abscess

a pus-containing lesion of the lung parenchyma that results


in a cavity formed by necrosis of lung tissue.

mechanical receptors

receptors located in lungs, upper airways, chest wall, and


diaphragm that are stimulated by irritants, muscle

Front

Back

stretching, and alveolar wall distortion.

metered-dose inhaler

aerosolized drug delivered in a specific amount by


activating the inhaler or by inhaling.

nasal polyps

benign mucous membrane masses that form slowly in


response to repeated inflammation of the sinus or nasal
mucosa and project into the nasal cavity.

normocapnia

normal arterial carbon dioxide pressure (PaCO2 35 to 45


mm Hg).

O2 toxicity

a condition of oxygen overdosage caused by prolonged


exposure to a high levels of oxygen; may inactivate
pulmonary surfactant and lead to development of acute
respiratory distress syndrome.

obstructive sleep apnea

a condition characterized by partial or complete upper


airway obstruction during sleep, causing apnea and
hypopnea.

pancreatic insufficiency

a condition characterized by inadequate production and


secretion of pancreatic hormones or enzymes.

Front

Back

panlobular emphysema

type of emphysema involving distention and destruction of


the entire primary respiratory lobule; usually associated
with "1-antitrypsin deficiency; also called chronic
hypertrophic, diffuse, generalized, panlobular, or vesicular
emphysema.

pleural effusion

an abnormal accumulation of fluid in the intrapleural spaces


of the lungs.

pleural friction rub

creaking or grating sound from roughened, inflamed


surfaces of the pleura rubbing together, evident during
inspiration, expiration, or both and no change with
coughing; usually uncomfortable, especially on deep
inspiration.

pleurisy (pleuritis)

inflammation of the pleura.

pneumoconiosis

a general term for lung diseases caused by inhalation and


retention of dust particles.

pneumonia

an acute inflammation of the lungs, often caused by inhaled


pneumococci of the species Streptococcus pneumoniae.

pneumothorax

a collection of air or gas in the pleural space causing the

Front

Back

lung to collapse.

postural drainage

the use of various positions to promote gravity drainage of


bronchial secretions; coughing usually expels secretions of
the trachea.

pulmonary edema

an abnormal accumulation of fluid in the alveoli and


interstitial spaces of the lungs caused most commonly by
heart failure; an acute, life-threatening situation in which the
lung alveoli become filled with serous or serosanguineous
fluid caused most commonly by heart failure.

pulmonary embolism

a thromboembolic occlusion of the pulmonary vasculature


resulting from thrombi in the venous circulation or right side
of the heart and from other sources, such as amniotic fluid,
air, fat, bone marrow, and foreign IV material that travel as
emboli until lodging in the pulmonary vessels.

pulmonary hypertension

elevated pulmonary pressure resulting from an increase in


pulmonary vascular resistance to blood flow through small
arteries and arterioles.

pursed-lip breathing

a technique of exhaling against pursed lips to prolong


exhalation, preventing bronchiolar collapse and air trapping;
done to increase expiratory airway pressure, improve
oxygenation of the blood, and help prevent early airway

Front

Back

closure.

rhinoplasty

the surgical reconstruction of the nose.

rhonchi

continuous rumbling, snoring, or rattling sounds from


obstruction of large airways with secretions; most
prominent on expiration; change often evident after
coughing or suctioning.

status asthmaticus

a severe, life-threatening asthma attack that is refractory to


usual treatment and places the patient at risk for developing
respiratory failure.

surfactant

a lipoprotein that lowers the surface tension in the alveoli,


reduces the amount of pressure needed to inflate the
alveoli, and decreases the tendency of the alveoli to
collapse.

tension pneumothorax

a pneumothorax with rapid accumulation of air in the pleural


space causing severely high intrapleural pressures with
resultant tension on the heart and great vessels.

thoracentesis

a surgical procedure done to remove fluid from the pleural


space.

Front

Back

thoracotomy

surgical opening into the thoracic cavity.

tidal volume

volume of air exchanged with each breath.

tracheostomy

surgical opening into the trachea through which an


indwelling tube may be inserted.

tracheotomy

a surgical incision into the trachea for the purpose of


establishing an airway; performed below a blockage by a
foreign body, tumor, or edema of the glottis.

trigger

a substance, object, or agent that initiates or stimulates an


action; in asthma, any stimuli that initiates the IgE

tuberculosis

an infectious disease caused by Mycobacterium


tuberculosis; usually involves the lungs but also occurs in
the larynx, kidneys, bones, adrenal glands, lymph nodes,
and meninges and can be disseminated throughout the
body.

vibration

pressing on the chest with the flat of the hands while


repeatedly tensing the hand and arm muscles to facilitate
movement of secretions to larger airways.

Front

Back

wheezes

a form of rhonchus characterized by continuous highpitched squeaking sound caused by rapid vibration of
bronchial walls.

Anda mungkin juga menyukai